Đến nội dung

AnhTran2911 nội dung

Có 230 mục bởi AnhTran2911 (Tìm giới hạn từ 07-05-2020)



Sắp theo                Sắp xếp  

#678272 Topic về Bất đẳng thức, cực trị THCS

Đã gửi bởi AnhTran2911 on 22-04-2017 - 00:12 trong Bất đẳng thức và cực trị

Nhờ mọi người làm giúp ạ:

Cho a,b,c dương thỏa mãn a+b+c=3

CMR : $\sum\frac{a}{b+c^2}\geq\frac{3}{2}$




#681715 Topic về Bất đẳng thức, cực trị THCS

Đã gửi bởi AnhTran2911 on 23-05-2017 - 22:25 trong Bất đẳng thức và cực trị

1. Cho a, b, c dương thỏa: 6a+3b+2c=abc. Tìm GTLN: $B=\frac{1}{\sqrt{a^{2}+1}}+\frac{2}{\sqrt{b^{2}+4}}+\frac{3}{\sqrt{c^{2}+9}}$

2. Cho các số a,b,c không âm. CMR: $a^{2}+b^{2}+c^{2}+3\sqrt[3]{(abc)^{2}} \geq 2(ab+bc+ca)$

Cái đầu chuyển từ $( a,b,c)\rightarrow(\frac{1}{a}; \frac{2}{b};\frac{3}{c})$ Đc bài toán quen thuộc

Cái thứ hai dùng Schur dạng pthức $\sum{a^2}+\frac{9abc}{a+b+c}\geq2\sum{ab}$




#678157 Topic về Bất đẳng thức, cực trị THCS

Đã gửi bởi AnhTran2911 on 20-04-2017 - 22:03 trong Bất đẳng thức và cực trị

bài 4 từ gt suy ra ab+bc+ca=abc
thay vào bài sigma a^2/(a+bc) = sigma a^3/(a^2+abc)= sig ma a^3/(a+b)(a+c) 
đến đây amgm 
a^3/(a+b)(a+c) + a+b/8 +b+c/8

Để tránh spam và làm mất đẹp topic, rất mong bạn sử dụng latex trong mỗi bài viết của mình. Thân.




#676848 Topic về Bất đẳng thức, cực trị THCS

Đã gửi bởi AnhTran2911 on 10-04-2017 - 00:06 trong Bất đẳng thức và cực trị


 

 

Đặt $(a+b-c;b+c-a;c+a-b)=(x;y;z) \Rightarrow \frac{x+y}{2}=b; \frac{y+z}{2}=c ;\frac{x+z}{2}=a$

Suy ra $BĐT \Leftrightarrow \frac{(x+y)(x+z)}{4x}+\frac{(y+z)(y+x)}{4y}+\frac{(z+x)(z+y)}{4z} \geq x+y+z$

                    $\Leftrightarrow (x+y)(y+z)(z+x)[\frac{1}{4x(y+z)}+\frac{1}{4y(x+z)}+\frac{1}{4z(x+y)}] \geq x+y+z$ $(*)$

Áp dụng bđt thân quen ta có: $(x+y)(y+z)(z+x) \geq \frac{8}{9}(x+y+z)(xy+yz+zx)$ $(1)$

Ap dụng bđt $Cauchy-Schwazt$ thì ta được $\frac{1}{4x(y+z)}+\frac{1}{4y(x+z)}+\frac{1}{4z(x+y)} \geq \frac{9}{8(xy+yz+zx)}$ $(2)$

Nhân $(1)$ với $(2)$ thì ta được $(x+y)(y+z)(z+x)[\frac{1}{4x(y+z)}+\frac{1}{4y(x+z)}+\frac{1}{4z(x+y)}]\geq  \frac{9}{8(xy+yz+zx)}.\frac{8}{9}(x+y+z)(xy+yz+zx)=x+y+z$ suy ra $(*)$ đúng $\Rightarrow ĐPCM$.

Đẳng thức xảy ra khi $a=b=c$.

liệu a, b, c thực thì có đúng k bạn




#676846 BĐT AM-GM

Đã gửi bởi AnhTran2911 on 09-04-2017 - 23:59 trong Bất đẳng thức và cực trị

Thách bạn nào làm được, làm được cho 1 Like :like  

Cho $ \left\{\begin{matrix} a,b,c>0 & \\ a+b+c=1& \end{matrix}\right.$

 

Tìm Min của S=$\frac{a^2+b}{b+c}+\frac{b^2+c}{c+a}+\frac{c^2+a}{a+b}$

S=$\frac{a^2+b}{b+c}+\frac{b^2+c}{c+a}+\frac{c^2+a}{a+b}$

  =$\sum$ $\frac{a^2}{b+c}+ $$\sum$ $\frac{b}{b+c}$=$\sum$ ($\frac{a^2}{b+c}+a)+$$\sum$ $\frac{b}{b+c}$-1

  =$\sum$ $\frac{a}{b+c} +$$\sum$ $\frac{b}{b+c}$ -1( do $\sum$a$ ={1}$)

  =$\sum$ $\frac{a+b}{b+c}-1 \geq 2$ ( Theo AM-GM cho 3 số dương).  KL. min biểu thức bằng 2.




#678269 [CHUYÊN ĐỀ] CHỨNG MINH BẤT ĐẲNG THỨC

Đã gửi bởi AnhTran2911 on 21-04-2017 - 23:54 trong Bất đẳng thức và cực trị

Solutions: Ta có : $A=\frac{\frac{1}{c}}{\frac{1}{a^{2}}+\frac{1}{b^{2}}}+\frac{\frac{1}{b}}{\frac{1}{a^{2}}+\frac{1}{c^{2}}}+\frac{\frac{1}{a}}{\frac{1}{c^{2}}+\frac{1}{b^{2}}}$

Đặt $(\frac{1}{a};\frac{1}{b};\frac{1}{c})=(x;y;z)$ khi đó $A=\frac{x}{y^{2}+z^{2}}+\frac{y}{x^{2}+z^{2}}+\frac{z}{y^{2}+x^{2}}$

                                                                                        $=\frac{x}{1-x^{2}}+\frac{y}{1-y^{2}}+\frac{z}{1-z^{2}}$ (*)

Ta có: $\frac{x}{1-x^{2}} \geq \frac{3\sqrt{3}}{2}x^{2}\Leftrightarrow (\sqrt{3}x-1)^{2}(\sqrt{3}x+2)\geq 0$ (1)

Dùng (1) cho (*) thì $A\geq \frac{3\sqrt{3}}{2}(x^{2}+y^{2}+z^{2})$

Mà $x^{2}+y^{2}+z^{2}=1$

Suy ra $A\geq \frac{3\sqrt{3}}{2}$

Vậy $Min A =  \frac{3\sqrt{3}}{2}$. DBXR khi $a=b=c=\sqrt{3}$.

Thay vì sd phương pháp tiếp tuyến như bạn Hoan ta cũng có thể sd đánh giá sau:

Sử dụng BĐT AMGM ta có : $2x^2(y^2+z^2)(y^2+z^2)\leq\frac{8}{27}$

$\rightarrow\sum\frac{x^4}{x^2(y^2+z^2)^2}\geq\frac{27x^4}{4}$

$\rightarrow\sum\frac{x}{y^2+z^2}\geq\frac{3\sqrt3x^{2}}{2}$

Lời giải đằng sau tương tự như bạn HOAN đã làm.




#678270 $\boxed{\text{Chuyên Đề}}$ Bất đẳng thức - Cực trị

Đã gửi bởi AnhTran2911 on 22-04-2017 - 00:08 trong Bất đẳng thức và cực trị

 

Bài 1: Cho a,b > 0 ; a+b=2 
Tìm GTNN: Q = $2(a^2+b^2)-6(\frac{a}{b}+\frac{b}{a})+9(\frac{1}{a^2}+\frac{1}{b^2})$

 

Bài 2: Cho các số dương x,y,z. Chứng minh BĐT: $\sqrt{\frac{x}{y+z}}+\sqrt{\frac{y}{x+z}}+\sqrt{\frac{z}{x+y}}>2$

 

Bài 3: Cho x,y $\epsilon$ R : $\sqrt{x-1}-y\sqrt{y} = \sqrt{y-1}-x\sqrt{x}$ 
Tìm GTNN S = $x^2+3xy-2y^2-8y+5$

 

Bài 4: Cho a,b,c > 0. CMR : 

 

$\frac{2ab}{3a+8b+6c}+\frac{3bc}{3b+6c+a}+\frac{3ca}{9c+4a+4b}\leq \frac{a+2b+3c}{9}$

 

Bài 5: Với x,y,z>0; xy+yz+zx=5 
Tìm GTNN: P= $\frac{3x+3y+2z}{\sqrt{6(x^2+5)} + \sqrt{6(y^2+5)} + \sqrt{z^2+5}}$

 

Bài 6: Cho các số thực dương a,b,c. Chứng minh rằng: 

 

$\sqrt{(a^2b+b^2c+c^2a)(ab^2+bc^2+ca^2)}\geq abc + \sqrt[3]{(a^3+abc)(b^3+abc)(c^3+abc)}$

 

Đẳng thức xảy ra khi nào? 

 

Bài 7: Cho a,b > 0; a+b = 1. Tìm GTNN 
T= $\frac{19}{ab}+\frac{6}{a^2+b^2}+2011(a^4+b^4)$

 

Bài1 và bài 7 bạn dùng phương chọn điểm rơi trong AM-GM hay bunhia đều đc

Bài 4 bạn đăt ẩn phụ a=x; 2b=y; 3c=z rồi dùng CS tách.

Bài 6 bạn chia 2 vế cho abc rồi tiến hành đặt ẩn phụ biểu thức 1 trong 2 căn đều đc rồi viết lại căn kia theo ẩn mới quy về BĐT 1 biến.

Đó là những hướng cm của mình.




#679398 Chuyên đề : Làm mạnh BĐT CôSy

Đã gửi bởi AnhTran2911 on 03-05-2017 - 21:39 trong Bất đẳng thức và cực trị

topic nát




#678465 Topic ôn thi hình học vào cấp 3 chuyên

Đã gửi bởi AnhTran2911 on 24-04-2017 - 11:28 trong Hình học

Lời giải bài 56.

                       attachicon.gifIMO 2006 1.png

Gọi giao điếm của $AQ,BQ$ với $DP,PC$ lần lượt tại $E,F$.

Dễ thấy vì $\frac{AM}{MB}=\frac{DN}{NC}$ nên các tia $DA,NM,CB$ đồng quy tại điểm $I$.

Áp dụng định lý $Menelaus$ cho tam giác $IAQ$ với 3 điểm $P,E,D$ thẳng hàng ta có:

                                      $\frac{QP}{IP}.\frac{DI}{AD}.\frac{AE}{EQ}=1$

Tương tự:

                                      $\frac{QP}{IP}.\frac{CI}{BC}.\frac{BF}{QF}=1$

                                      $\frac{DI}{AD}=\frac{CI}{BC}$  $(AB//CD)$

Nên $\frac{AE}{EQ}=\frac{BF}{QF}$ hay $EF//AB/CD$

Ta có: $\widehat{EBF}+\widehat{EQF}=\widehat{ABC}+\widehat{ACB}=180^{\circ}$

          $\Rightarrow$ Tứ giác $EPFQ$ nội tiếp   $(1)$

Suy ra $\widehat{EPF}=\widehat{EPQ}+\widehat{PQF}=\widehat{EFQ}+\widehat{QPF}=\widehat{ABQ}+\widehat{QPF}$

Mặt khác $\widehat{EPF}=\widehat{ABC}=\widehat{ABQ}+\widehat{QBC}$

   $\Rightarrow \widehat{QPF}=\widehat{QBC}$

   $\Rightarrow$ Tứ giác $PBCQ$ nội tiếp $\boxed{}$.

Thế còn Trường hợp Q nằm giữa P và M thì sao? bởi vì lúc này tứ giác EPFQ k nội tiếp




#678381 Topic ôn thi hình học vào cấp 3 chuyên

Đã gửi bởi AnhTran2911 on 23-04-2017 - 11:45 trong Hình học

Thông cảm vì mình không biết vẽ hình.

Lấy điểm Z sao cho I là trung điểm HZ. Do đó tứ giác AHCZ là hbh.

Ta có: EH/ZC=EH\AH=EL\LC ( Do EAB$\sim$ECD )

$\Rightarrow$ EH\EL=CZ\CL (1)

Lại có: $\angle$ LCZ = $\angle$ACZ-$\angle$ACD= $\angle$BAC-$\angle$ABD= 1/2 ( $\angle$BOC-$\angle$AOD) (2)

$\angle$LEH =$\angle$DEC-2$\angle$DEL=2$\angle$ADC-$\angle$DEC -180

                      = $\angle$AOC+ 1\2($\angle$DOC-$\angle$AOB)-180 (3)

Từ (2)(3) ta suy ra $\angle$LEH = $\angle$LCZ ( Biến đổi tương đương đương)(4)

Vì thế từ(1)(4)$\Rightarrow$ $\triangle$LEH đồng dạng $\triangle$LCZ . Dẫn đến $\angle$ELH =$\angle$CLZ

Cho nên $\angle$HLZ =90 $\Rightarrow$ $\triangle$HlZ vuông tại L. Mà $\triangle$ này có LI trung tuyến 

Suy ra IL=IH , CMTT thì JH=JL Nên JI là trung trực HL




#680390 Topic BẤT ĐẲNG THỨC ôn thi vào lớp 10 THPT 2017 - 2018

Đã gửi bởi AnhTran2911 on 12-05-2017 - 14:54 trong Bất đẳng thức và cực trị

BÀI 55 (sưu tầm)

Cho x,y,z là 3 số dương thỏa mãn: 1+x+y+z=2xyz

Tìm GTNN của P= $\frac{xy}{1+x+y}+\frac{yz}{1+y+z}+\frac{zx}{1+z+x}$ 

Đặt $(a,b,c)=(\frac{1}{x},\frac{1}{y},\frac{1}{z})$ Giả thiết viết lại thành : $ ab+bc+ca+abc=2$ 

Hay $ \sum\frac{1}{(a+1)(b+1)} =1$ $(1)$

Ta cần tìm min của $\sum\frac{1}{ab+a+b}$

Theo CS ta có $\frac{9}{ab+a+b}+1\geq{\frac{16}{(a+1)(b+1)}}$

Xây dựng các BĐT tương tư rồi dùng giả thiết $(1)$ ta xác định được Min




#679702 Topic BẤT ĐẲNG THỨC ôn thi vào lớp 10 THPT 2017 - 2018

Đã gửi bởi AnhTran2911 on 06-05-2017 - 15:50 trong Bất đẳng thức và cực trị

Bái Toán 51 ( VÕ QUỐC BÁ CẨN )

Cho $a,b,c \geq0$ thỏa mãn không có hai số nào đồng thời bằng 0 

CMR; $\sum\frac{1}{\sqrt{3a^2+bc}}\geq\frac{2+\sqrt3}{\sqrt{3(ab+bc+ca)}}$




#681728 Topic BẤT ĐẲNG THỨC ôn thi vào lớp 10 THPT 2017 - 2018

Đã gửi bởi AnhTran2911 on 23-05-2017 - 23:14 trong Bất đẳng thức và cực trị

Vấn đề 88: Cho a,b,c thực dương.

CMR: $\sum{\sqrt{(a+b)(a+c)}} \geq a+b+c + \sqrt{3(ab+bc+ca)}$

 




#680396 Topic BẤT ĐẲNG THỨC ôn thi vào lớp 10 THPT 2017 - 2018

Đã gửi bởi AnhTran2911 on 12-05-2017 - 15:39 trong Bất đẳng thức và cực trị

Bài Toán 56. (Lê Việt Hưng - Mr CooperCho $a,b,c$ là các số thực không âm thỏa $a^2+b^2+c^2=3$. Chứng minh rằng:

\[a \sqrt{b^2+c^2} + b \sqrt{c^2+a^2} + c \sqrt{a^2+b^2} \le \sqrt{6(a+b+c)} \]

Lời giải bài 56: Theo CS thì:

Ta có $(a^2b+b^2c+c^2a)\leq\sqrt{\frac{(a^2+b^2+c^2)^3}{3}}=3$

CM tương tự thì ta suy ra $\sum{ab(a+b)}\leq6$ $(1)$

Theo CS tiếp và $(1)$ thì $\sum(a(\sqrt{b^2+c^2})\leq\sqrt{(a+b+c)(\sum{ab(a+b)})}\leq VP$

Đến đây ta có ĐPCM.




#680399 Topic BẤT ĐẲNG THỨC ôn thi vào lớp 10 THPT 2017 - 2018

Đã gửi bởi AnhTran2911 on 12-05-2017 - 15:59 trong Bất đẳng thức và cực trị

Bài toán 57(sưu tầm)

CMR với mọi số a,b,c dương ta có:

$\frac{4(a^{2}+b^{2}+c^{2})}{ab+bc+ca}+\sqrt{3}\left ( \frac{a+2b}{\sqrt{a^{2}+2b^{2}}}+\frac{b+2c}{\sqrt{b^{2}+2c^{2}}}+\frac{c+2a}{\sqrt{c^{2}+2a^{2}}} \right )\geq 13$

Theo AM-GM và CS thì:

$\sum$ $\frac{3a+6b}{\sqrt{3a^2+6b^2}}$ = $\sum\frac{3(a+2b)^2}{(a+2b)(\sqrt{3(a^2+2b^2)}}$

$\geq\frac{3(a+2b)^2}{2a^2+2ab+5b^2}\geq\frac{27(a+b+c)^2}{7(a^2+b^2+c^2)+2(ab+bc+ca)}$

Đến đây đặt  $ ab+bc+ca=y ; a^2+b^2+c^2= x$

Ta đi CM : $\frac{4x}{y}+\frac{27(x+2y)}{7x+2y}\geq13$

Đương nhiên đúng vì nó tđg với $(x-y)^2\geq0$ ĐPCM.




#681725 Topic BẤT ĐẲNG THỨC ôn thi vào lớp 10 THPT 2017 - 2018

Đã gửi bởi AnhTran2911 on 23-05-2017 - 23:07 trong Bất đẳng thức và cực trị

Bài toán 87(sưu tầm)

Cho x,y thỏa mãn $0< x,y\leqslant 1$ và x+y=3xy. Tìm GTLN và GTNN của

biểu thức P= $x^{2}+y^

Ta có:

P=$9x^2y^2-6xy$

Cần tìm min max $3x^2y^2-2xy$

Max: từ giả thiết $\rightarrow(xy-1)(3xy+1)\leq0\rightarrow 3x^2y^2-2xy\leq1$

Min : Từ $ x+y=3xy\rightarrow xy\geq\frac{4}{9}$

Nếu $xy\geq\frac{2}{3} : P\geq 0$ 

Nếu $xy\leq\frac{2}{3} : 2-3xy\geq0$ nên áp dung cauchy 3 số :$ xy(2-3xy)\leq\frac{(x+y+2-3xy)^3}{27}=\frac{8}{27}$

        Nên lúc này $3x^2y^2-2xy\geq\frac{-8}{27}\Rightarrow P\geq\frac{-8}{27}$

So sánh KL $minP=\frac{-8}{27} \leftrightarrow x=y=\frac{2}{3}$




#679701 Topic BẤT ĐẲNG THỨC ôn thi vào lớp 10 THPT 2017 - 2018

Đã gửi bởi AnhTran2911 on 06-05-2017 - 15:28 trong Bất đẳng thức và cực trị

Bài toán 50 (HELLO IMO 2007).

Cho các số thực không âm $a,b,c$. Chứng minh rằng

                                  $abc+2(a^{2}+b^{2}+c^{2})+8 \geq 5(a+b+c)$.

$LHS-RHS=\frac{1}{2} ((a^2+b^2+c^2+2abc+1-2ab-2bc-2ca)+(a+b+c-3)^2+2\sum(a-1)^2)\geq0$

PPS: Đây là hệ quả trực tiếp của BĐT tổng quát thầy Trần Nam Dũng:

Cho $a,b,c \geq0$ thì $abc+2+k[\sum(a-1)^2]\geq{a+b+c}$ Với $k$ là số thực, $k\geq\frac{1}{\sqrt2}$




#679120 Topic BẤT ĐẲNG THỨC ôn thi vào lớp 10 THPT 2017 - 2018

Đã gửi bởi AnhTran2911 on 01-05-2017 - 14:49 trong Bất đẳng thức và cực trị

$\boxed{\textbf{ BÀI TOÁN 31 }}$ 

Cho $ a,b,c\ge 0$ . Chứng Minh Rằng:

$3(a^2+b^2+c^2)(a^2b^2+b^2c^2+c^2a^2)\geq(a^2+ab+b^2)(b^2+bc+c^2)(c^2+ca+a^2)$




#678870 Topic BẤT ĐẲNG THỨC ôn thi vào lớp 10 THPT 2017 - 2018

Đã gửi bởi AnhTran2911 on 28-04-2017 - 22:29 trong Bất đẳng thức và cực trị

Bài toán làm mạnh một bất đẳng thức quen thuộc

 

$\boxed{\textbf{Bài Toán 26}}$ $\text{[Lê Việt Hưng - Mr Cooper]} $ Chứng minh bất đẳng thức sau đúng với mọi a,b,c. 

\[a^2+b^2+c^2 \ge \sum \dfrac{a(b^2+c^2)}{b+c} \ge ab+bc+ca\]

CM vế đầu tiên:

BĐT đã cho tương đương:

$\sum{a^2}\geq a(b+c) - \frac{2abc}{b+c}$

$\Rightarrow\sum{a^2}+ 2abc(\sum\frac{1}{b+c})\geq 2\sum{ab}$

Áp dụng CS ta CM : $\sum{a^2} + \frac{9abc}{a+b+c}\geq 2\sum{ab}$

Đây là Schur phân thức.




#678121 Topic BẤT ĐẲNG THỨC ôn thi vào lớp 10 THPT 2017 - 2018

Đã gửi bởi AnhTran2911 on 20-04-2017 - 17:32 trong Bất đẳng thức và cực trị

Đề xuất bài tiếp theo:

 

Bài 19 (Tạp chí TTT):

Cho các số thực không âm thỏa mãn $a^{2}+b^{2}+c^{2} \not = 0$

Tìm giá trị nhỏ nhất của biểu thức

                                                             $P=\frac{\sqrt{a+2b+3c}}{\sqrt{2a+2b+3c}+\sqrt{a+5b+3c}+\sqrt{a+2b+7c}}$

 

Bài 20 (Tạp chí TTT):

Cho $a,b,c$ là các số thực dương .Chứng minh rằng:

                                                              $a^{2}+b^{2}+c^{2}\geq \frac{9abc}{a+b+c}+(c-a)^{2}$

 

Bài 21 (Tạp chí AMM):

 Cho các số $a,b,c \in [1;2]$. Chứng minh rằng:

                                                              $(3a+2b+c)(\frac{1}{a}+\frac{1}{b}+\frac{1}{c})\leq \frac{45}{2}$

Dấu bằng xảy ra khi nào ? 

2 bài đầu có trong mục chữa bài ở báo .Xin góp ý cho bài toàn cuối:

Ta có Bổ đề quen thuộc sau: Với $a,b,c \in [1;2]$. CM: $(a+b+c)(\frac{1}{a}+\frac{1}{b}+\frac{1}{c})\leq10$

Bổ đề trên các bạn tham khảo ở NCPT toán 9 tập 1.

Áp dụng bất đẳng thức trên ta quy bài toán về cm : $(2a+b)(\frac{1}{a}+\frac{1}{b}+\frac{1}{c})\leq\frac{25}{2}$

 

Khai triển và sử dụng gt c>=1 ta có: $\frac{2a}{b} +\frac{b}{a} +2a+b \leq\frac{19}{2}$

BĐT này tương đương với : $(4+4b)a^{2} +a(2b^{2}-19b) +2b^{2}\leq 0$

Nhân thấy đây là tam thức d2, tham số b. Hệ số cao nhất dương . a thuộc khoảng [1,2 ] nên theo định lý về hàm cực trị biên thì :

$f(a)\leq max({f(1);f(2))}$ . Đến đây thì đơn giản rồi ta xét BĐT trên tại a=1 và a=2 để từ đó CM vs BĐT 1 biến b với ĐK b thuộc [1,2]. CM cái này không mấy khó khăn, các bạn hãy thử xem.




#678903 Topic BẤT ĐẲNG THỨC ôn thi vào lớp 10 THPT 2017 - 2018

Đã gửi bởi AnhTran2911 on 29-04-2017 - 12:59 trong Bất đẳng thức và cực trị

$\boxed{ BÀI TOÁN 27}$ ( ĐẠI HỌC KHỐI B 2010):

Cho $a,b,c\geq0$ thỏa mãn $a+b+c=1$

Tìm Min $3(a^2b^2+b^2c^2+c^2a^2)+3(ab+bc+ca)+2\sqrt{a^2+b^2+c^2}$




#679106 Topic BẤT ĐẲNG THỨC ôn thi vào lớp 10 THPT 2017 - 2018

Đã gửi bởi AnhTran2911 on 01-05-2017 - 11:49 trong Bất đẳng thức và cực trị

$\boxed{\textbf{Bài Toán 28}} $

Cho các số a,b,c không âm thoả mãn:$a^2+b^2+c^2=2(ab+bc+ca)$.Chứng minh rằng:

$\sqrt{1+\frac{a}{b+c}}+\sqrt{1+\frac{b}{c+a}}+\sqrt{1+\frac{c}{a+b}}\geq 1+2\sqrt{2}$

Bài này tương đối khó .

Chuẩn hóa $a+b+c=2$. GT có thể viết lại thành $ab+bc+ca=1$.

BĐT cần CM tương đương với : $\sum\frac{1}{\sqrt{a+b}}\geq2+\frac{1}{\sqrt2}$

Đến đây ta có BĐT sau ( Với gs a max) : $\frac{1}{\sqrt{b^2+1}}+\frac{1}{\sqrt{c^2+1}}\geq1+\frac{1}{\sqrt{(b+c)^2+1}}$

( CM cái này các bạn bình phương lên là đc, quá dài mình k trình bày ở đây)

Từ đây suy ra $\frac{1}{\sqrt{a+b}}+\frac{1}{\sqrt{a+c}} \geq\sqrt{b+c}+ \sqrt {\frac{b+c}{(b+c)^2+1}}$

Đến đây quy hàm một biến $t=\sqrt{b+c}+\frac{1}{\sqrt{b+c}} ( t\geq2) $

CM BĐT 1 biến cuối cùng > Đơn giản chỉ là 1 AMGM quen thuộc.




#679448 Topic BẤT ĐẲNG THỨC ôn thi vào lớp 10 THPT 2017 - 2018

Đã gửi bởi AnhTran2911 on 04-05-2017 - 11:19 trong Bất đẳng thức và cực trị

$\boxed{\textbf{Bài Toán 45}}$ $\text{[IMO 2001]}$ Cho $a,b,c$ là các số thực dương. Chứng minh rằng:

\[\frac{a}{\sqrt{a^2+8bc}}+\frac{b}{\sqrt{b^2+8ca}}+\frac{c}{\sqrt{c^2+8ab}} \ge 1\]

Theo CS thì $LHS=\sum\frac{a^2}{a\sqrt{a^2+8bc}}\geq\frac{(a+b+c)^2}{\sum{a}\sqrt{a^2+8bc}}\geq$

                           $\frac{(a+b+c)^2}{\sqrt{(a+b+c)(\sum{a^3}+24abc)}}\geq$ $\frac{(a+b+c)^2}{\sqrt{(a+b+c)(a+b+c)^3}}=RHS$




#679295 Topic BẤT ĐẲNG THỨC ôn thi vào lớp 10 THPT 2017 - 2018

Đã gửi bởi AnhTran2911 on 02-05-2017 - 22:07 trong Bất đẳng thức và cực trị

Dễ có $a,b,c\leq\frac{1}{2}$ . Do vậy $(1-2a)(1-2b)(1-2c)\geq0$ ( do k hông xảy ra TH cả 3 số đều bằng $\frac{1}{2}$)

$\Rightarrow 1-8abc-2(a+b+c) +4(ab+bc+ca)\ge0 = 1-8abc-2(a+b+c)^2+4(ab+bc+ca)\ge0$

:ukliam2: CM vế phải: sử dụng BĐT AM-GM cho 3 số dương ta có:

$(1-2a)(1-2b)(1-2c)\leq\frac{1}{27}$ ( Do $a+b+c=1$)

$\Rightarrow-8abc+4(ab+bc+ca)-2(a+b+c)\leq\frac{-26}{27}$

$\Rightarrow4abc+a+b+c-2(ab+bc+ca)\geq\frac{13}{27}$

$\Rightarrow4abc+(a+b+c)^2-2(ab+bc+ca)\geq\frac{13}{27}$

$\Rightarrow4abc+a^2+b^2+c^2\geq\frac{13}{27}$. ( ĐPCM).




#679640 Topic BẤT ĐẲNG THỨC ôn thi vào lớp 10 THPT 2017 - 2018

Đã gửi bởi AnhTran2911 on 05-05-2017 - 23:13 trong Bất đẳng thức và cực trị

BÀI 47:  (ĐỀ TOÁN CHUYÊN HCM-15-16) 

 Cho 2 số thực dương a,b thỏa mãn điều kiện: $a+b\leq 1$

CMR: $a^{2}-\frac{3}{4a}-\frac{a}{b}\leq -\frac{9}{4}$

BĐT $\Leftrightarrow\frac{a}{b}+\frac{3}{4a}\geq{a^2}+\frac{9}{4}$

Do $a+b\leq1$ nên $4ab\leq1$ và ta có đgiá $\frac{a}{b}\geq4a-4ab\geq4a-1$

Đưa về BĐT $4a+\frac{3}{4a}\geq{a^2}+\frac{13}{4}$ Đúng vì nó tương đương với $(3-a)(2a-1)^2\geq0$

Ps: Có ai làm được bài số 46 của bạn Nhoang1608 chưa ạ. Cho hỏi cái dấu bằng :v